Drug company manager: Our newest product is just not selling. One way to save it would be a new marketing campaign. T...

iris.diaz823 on July 12, 2020

Not B

I was torn between B and D. Ultimately, I chose D because B seems too general. it talks about all new products and all marketing campaigns instead of the new products in the drug company and new marketing campaigns. Is this correct reasoning?

Reply
Create a free account to read and take part in forum discussions.

Already have an account? log in

shunhe on July 13, 2020

Hi @iris.diaz823,

Thanks for the question! So I would say that (B) is wrong for a couple of reasons, including some of the ones that you mentioned. Mainly, it just tells us that many new products fail, whether or not they’re supported by marketing campaigns. Does this weaken the manager’s argument that we should try the marketing campaign? No, not really. And also, remember that “many” is pretty similar to “some” on the LSAT in that it basically means at least one. But it doesn’t tell us how many “many” is. It could be 2 products, it could be 2345 products. So we just don’t know enough from (B) for it to really weaken the argument.

(D), on the other hand, gives us a strong reason to not try the new marketing campaign. The drug company manager is basically saying “well, this might not work, but hey let’s try it anyway.” And (D) says, “oh wait, but also we should be wary about just trying it because trying it might be bad for us in other ways.” And that weakens the drug company manager’s argument that we should try the new marketing campaign, since they didn’t consider all the possible implications.

Hope this helps! Feel free to ask any other questions that you might have.